Intégrale mercredi 3 août

2»

Réponses

  • J'ai oublié de donner un lien qui démontre Frullani sans intégrale double https://9to5science.com/proof-of-frullani-39-s-theorem
    Le 😄 Farceur


  • etanche
    Modifié (September 2022)
    La preuve 1 du lien ci-dessus était dans le livre exercices d’analyse de Lucien Chambadal 
    https://lmbp.uca.fr/biblio/notice.php?num=7614

  • @Gebrane:  Pour appliquer Frullani à $\displaystyle \int_0^1 \frac{z-1}{(1+z)\ln z}dz$ Le changement de variable $u=-\ln z$ doit aider, on fait apparaître un facteur $\dfrac{1}{u}$ et on a les bonnes bornes.
  • On a donc
    \begin{align}\int_0^1 \frac{z-1}{(1+z)\ln z}dz&\overset{u=-\ln z}=\int_0^\infty \frac{\text{e}^{-u}-\text{e}^{-2u}}{\left(1+\text{e}^{-u}\right)}\times \frac{1}{u}du\end{align} Est-ce qu'il existe une fonction $f$ et des réels $a,b$ tels que $\displaystyle \frac{\text{e}^{-u}-\text{e}^{-2u}}{\left(1+\text{e}^{-u}\right)}=f(au)-f(bu)$? L'intégrale proposée a pour valeur $\ln\left(\dfrac{\pi}{2}\right)$ ce qui suggère que $a$ ou $b$ vaudrait $\pi$. Bof.
  • Fin de partie
    Modifié (September 2022)
    Comment une fraction rationnelle d'exponentielles réelles pourrait avoir en $0$ ou en l'infini une limite qui vaut $\pi$?
    On va devoir évaluer une fonction $f$ en $c\pi u$ et en $b u$ avec $\dfrac{c\pi}{b}=\dfrac{2}{\pi}$ (ou l'inverse) et $\displaystyle f(c\pi u)-f(b u)=\frac{\text{e}^{-u}-\text{e}^{-2u}}{\left(1+\text{e}^{-u}\right)}$.
  • @Fin de partie  Utilise le développement en série de $1/(1+e^{-u})$, puis intervertis Somme/Intégrales. Tu tomberas sur une belle somme à trouver sa somme
    Le 😄 Farceur


  • Fin de partie
    Modifié (September 2022)
    @Gebrane: Calculer ton intégrale peut se faire par la même méthode que celle déjà utilisée pour l'intégrale de l'AMM.
    Je cherche comment calculer cette intégrale avec la formule de Frullani comme tu l'as demandé.
    PS.
    Cela ne marche pas avec au dénominateur $1+x$ il faut impérativement que l'exposant de $1+x$ soit $3$.
  • gebrane
    Modifié (September 2022)
    Bah dans ce lienhttps://les-mathematiques.net/vanilla/index.php?p=/discussion/comment/2380233/#Comment_2380233  je t'aide à comprendre comment utiliser Frullani dans le cas simple de $\int_0^1 \frac{z-1}{(1+z)\ln z}dz$.

    Utiliser Frullani est une richesse d'idées. 
    Le 😄 Farceur


  • gebrane
    Modifié (September 2022)
    Je me répète pour appliquer Frullani on a besoin  de développer en série (expliqué dans ce messagehttps://les-mathematiques.net/vanilla/index.php?p=/discussion/comment/2380233/#Comment_2380233
    Le 😄 Farceur


  • gebrane
    Modifié (September 2022)
    Je te dis ça marche pour $\int_0^1 \frac{z-1}{(1+z)\ln z}dz$, mais pour l'exposant 3 je tombe sur une bizarrerie un truc divergent.
    Le 😄 Farceur


  • @Gebrane: Tu peux me donner la fonction $f$ utilisée pour appliquer la formule de Frullani?
  • Tu tombes sur ceci 
    $$\sum_{n=0}^{\infty} (-1)^n \int_0^{\infty} \frac{ (e^{-(n+1) u} - e^{-(n+2) u})}{u}  \, du$$
    Le 😄 Farceur


  • @Gebrane: Je m'étais imaginé que Frullani permettait d'avoir tout de suite la valeur de l'intégrale. >:)
  • FDP je parle d'une richesse de méthodes . Avec Frullani on tombe comme dans ton lien sur 
    $$\sum_{n=0}^{+\infty} (-1)^n \ln(\frac{n+1}{n+2})$$
    Le 😄 Farceur


  • @Gebrane: la suite du calcul est moche mais c'est une appréciation personnelle.
  • J'ai espoir qu'on arrive à utiliser que $\Psi(x)=\dfrac{d}{dx} \ln\left(\Gamma(x)\right)=\dfrac{\Gamma^\prime (x)}{\Gamma(x)}$

  • gebrane
    Modifié (September 2022)
    Dans le cas de la puissance 3  et en faisant le développent de Taylor de 1(1+x)^3 je tombe si j'intervertis Somme et Intégrale  sur la  série  $\sum_{n=0}^{+\infty} (-1)^n (n+1)(n+2)\ln(\frac{n+1}{n+2})$ divergente 

    L'arnaque c'est que wolfram dit par exemple que la série  $\sum_{n=0}^{+\infty} (-1)^n n^2 \ln(n)$  diverge $https://www.wolframalpha.com/input?i=\sum_{n=0}^{+\infty}+(-1)^n+n^2+\ln(n)$ mais wiki  https://en.wikipedia.org/wiki/Dirichlet_eta_function  
    donne un sens à cette série c'est $\eta'(-2)$ (un truc de prolongement analytique)
    Le 😄 Farceur


  • On a $\displaystyle  \int_{\frac{1}{2}}^3 \Psi(x)dx=\ln 2-\frac{1}{2}\ln(\pi)$ et on a la formule $\displaystyle \Psi(s+1)=-\gamma+\int_0^1 \frac{1-x^s}{1-x}dx$ on devrait arriver à en faire quelque chose.

  • $\displaystyle \int_0^1 \frac{z-1}{(1+z)^3\ln z}dz=\dfrac{7\zeta(3)}{4\pi^2}$
  • Fin de partie
    Modifié (September 2022)
    C'est nettement plus joli
    \begin{align*}
    J&=\int_0^1 \frac{z-1}{(1+z)\ln z}dz\\
    &=\int_0^1 \int_0^1 \frac{x^t}{1+x}dtdx\\
    &\int_0^1 \int_0^1 \frac{x^t-x^{t+1}}{1-x^2}dtdx\\
    &\overset{u=x^2}=\frac{1}{2}\int_0^1\int_0^1 \frac{u^{\frac{t-1}{2}}-u^{\frac{t}{2}}}{1-u}du\\
    &=\frac{1}{2}\int_0^1\bigg(\underbrace{\int_0^1 \frac{1-u^{\frac{t}{2}}}{1-u}du}_{=\Psi\left(1+\frac{t}{2}\right)+\gamma}-\underbrace{\int_0^1 \frac{1-u^{\frac{t-1}{2}}}{1-u}du}_{=\Psi\left(\frac{t-1}{2}\right)+\gamma}\bigg)\\
    &=\frac{1}{2}\int_0^1 \left(\Psi\left(1+\frac{t}{2}\right)-\Psi\left(\frac{1+t}{2}\right)\right)dt\\
    &=\left[\ln\left(\frac{\Gamma\left(1+\frac{u}{2}\right)}{\Gamma\left(\frac{1+u}{2}\right)}\right)\right]_0^1\\
    &=\ln\Big(\frac{\Gamma\left(\frac{3}{2}\right)}{\Gamma\left(1\right)}\Big)-\ln\Big(\frac{\Gamma\left(1\right)}{\Gamma\left(\frac{1}{2}\right)}\Big)\\
    &=\ln\Big(\underbrace{\Gamma\Big(\frac{3}{2}\Big)}_{\frac{1}{2}\Gamma\left(\frac{1}{2}\right)}\Big)+\ln\Big(\underbrace{\Gamma\Big(\frac{1}{2}\Big)}_{\sqrt{\pi}}\Big)\\
    &=\boxed{\ln\left(\frac{\pi}{2}\right)}
    \end{align*}
  • C'est joli  
    Le 😄 Farceur


  • Est-il possible de démontrer la formule $\displaystyle\Psi(s+1)=-\gamma+\int_0^1 \frac{1-x^s}{1-x}dx$
    sans utiliser l'expression en produit infini de $\dfrac{1}{\Gamma(x)}$?
    avec:$\displaystyle \Psi(x)=\dfrac{d}{dx} \ln\left(\Gamma(x)\right)=\dfrac{\Gamma^\prime (x)}{\Gamma(x)}$
  • Fin de partie
    Modifié (September 2022)
    @Gebrane: Si je lis bien, ce que tu indiques utilise le produit infini de $\dfrac{1}{\Gamma(x)}$.
    L'utilisation du théorème de Frullani amène à considérer une série (qui est calculée avec le produit infini susmentionné).
    PS.
    On montre que \begin{align}H_n:=\sum_{k=1}^n \frac{1}{k}=\int_0^1\frac{1-x^n}{1-x}dx\end{align}
    Comme la précédente intégrale existe pour n'importe quel réel $n>-1$ on peut parler de $H_s$ pour $s$ un réel $>-1$.
    Robjohn utilise la formule que je veux démontrer. >:)
  • Fin de partie
    Modifié (September 2022)
    On a : \begin{align}\gamma&=\int_0^1 \left(\frac{1}{\ln x}+\frac{1}{1-x}\right)\\ \Gamma(t)&=\int_0^1 (-\ln x)^{t-1}dx \end{align} Démontrer la formule : \begin{align}\Psi(s+1)=-\gamma+\int_0^1 \frac{1-x^t}{1-x}dx\end{align} revient à démontrer : \begin{align}\Gamma^\prime(t+1)=-\int_0^1\int_0^1 (-\ln x)^t\left(\frac{1}{\ln y}+\frac{y^t}{1-y}\right)dxdy \end{align}
    NB. Je ne sais pas démontrer la première formule qui donne $\gamma$ tout de suite maintenant mais je doute que cette démonstration repose sur le produit infini de $\dfrac{1}{\Gamma(x)}$. Je sors cette formule du livre de Julian Havil, Gamma,Exploring Euler's constant, édité par Princeton, 2003
    PS.
    J'avais oublié $y^t$. Corrigé.
  • gebrane
    Modifié (September 2022)
    @Fin de partie Si je te comprends bien tu veux démontrer pour commencer que 
    $$\gamma=\int_0^1 \left(\frac{1}{\ln x}+\frac{1}{1-x}\right)$$ C'est ça ?
    Le 😄 Farceur


  • @Gebrane: Je suis sûr qu'il y a un post sur MathExchange qui donne une preuve de cette formule.
  • gebrane
    Modifié (September 2022)
    @Fin de partie  Pas besoin d'un lien. On part de $\gamma$ est la limite de $H_n-\ln(n)$ et on met sous forme intégrale pour voir.
    Le 😄 Farceur


  • gebrane
    Modifié (September 2022)
    On a $H_n=\sum_{k=1}^n \frac 1k=\sum_{k=1}^n \int_0^1 x^{k-1} dx=\int_0^1 \sum_{k=1}^n x^{k-1}=\int_0^1 \frac {1-x^n}{1-x} dx$
    donc $\gamma$ est limite de $\int_0^1 \frac {1-x^n}{1-x} dx-\int_1^n \frac 1x  dx$  à méditer pour la suite si 


    En suite on pose x=1-t/n et on utilise le TCD et on tombe sur deux intégrales avec des exponentiels car  $(1-x/n)^n$ tend vers $e^{-x}$

    donc ok pour la formule 1
    Le 😄 Farceur


  • Fin de partie
    Modifié (September 2022)
    @Gebrane. Le truc qui a toute mon attention c'est de démontrer :
    \begin{align}\underbrace{\int_0^\infty x^{t}\text{e}^{-x}\ln xdx}_{=\Gamma^\prime (t+1)}=-\int_0^1\int_0^1 (-\ln x)^t\left(\frac{1}{\ln y}+\frac{y^t}{1-y}\right)dxdy\end{align}
    PS.
    Évidemment sans utilisation du produit infini de $\dfrac{1}{\Gamma(x)}$.
  • gebrane
    Modifié (September 2022)
    Bonjour @Fin de partie     Je ne comprends pas pourquoi tu veux éviter l'utilisation de la formule 
    $\Gamma(z+1) = e^{-\gamma z}\prod_{n\geq 1}\left(1+\frac{z}{n}\right)^{-1}e^{z/n}.$ sa démonstration est simple et m'a donné l'idée pour démontrer ta première formule 
    $\gamma=\int_0^1 \left(\frac{1}{\ln x}+\frac{1}{1-x}\right)$
    Le 😄 Farceur


  • Allo !
    Le 😄 Farceur


  • Fin de partie
    Modifié (September 2022)
    @Gebrane: Je n'ai pas le droit d'avoir mes marottes ? >:) Pas sûr du tout qu'on puisse trouver un moyen de calculer cette intégrale double sans l'aide de ce produit infini même si j'espère que c'est possible.
    PS.
    Un type* a écrit un bouquin en se privant de l'utilisation de la lettre e, qu'il a titré justement La disparition.
    Je trouve ça chouette ce genre de défi qu'on se lance.
    *Georges Perec
  • gebrane
    Modifié (September 2022)
    En regardant cette video 
    est-ce que ça te donne une idée pour démontrer ta première formule, il y a le TCD à appliquer.
    je subis un massacre.
    Le 😄 Farceur


  • Fin de partie
    Modifié (September 2022)
    Je travaille sur une démonstration de $\displaystyle \gamma=\int_0^1 \left(\frac{1}{\ln x}+\frac{1}{1-x}\right)dx$
    mais je butte sur la démonstration de $\displaystyle \lim_{n\rightarrow \infty}\int_{\frac{1}{n}}^1\left(\frac{1}{\ln x}+\frac{x^n}{1-x}+\frac{n}{1+nx}\right)dx=-\ln 2$
    PS.
    La démonstration de $\displaystyle \lim_{n\rightarrow \infty}\int_0^{\frac{1}{n}}\left(\frac{1}{\ln x}+\frac{x^n}{1-x}+\frac{n}{1+nx}\right)dx=\ln 2$
    est nettement plus simple.
    PS2.
    Bien évidemment, je cherche une démonstration sans relation avec une formule avec $\gamma$.
    PS3.
    J'ai obtenu au départ ces deux limites empiriquement.
    f(x,n)={1/log(x)+x^n/(1-x)+n/(1+n*x)};
    j(n)={intnum(x=0,1/n,f(x,n))}
    k(n)={intnum(x=1/n,1,f(x,n))}
    print(j(1000000)," ",k(1000000)) 0.6931471127875097431210104166 -0.6931465701753139465334581215
  • Fin de partie
    Modifié (September 2022)
    Pour le contrôle de $\dfrac{1}{\ln x}$ , j'imagine que $\displaystyle \lim_{x\rightarrow 1} \left(\frac{1}{\ln x}+\frac{1}{1-x}\right)=\frac{1}{2}$ aide.
    PS.
    Erreur de signe corrigée.
  • La vidéo m'a suggerée une preuve de $\displaystyle \gamma=\int_0^1 \left(\frac{1}{\ln x}+\frac{1}{1-x}\right)dx$
    Je pars de $\gamma=\lim_{n\to \infty} \int_0^1 \frac {1-x^n}{1-x} dx-\int_1^n \frac 1x  dx$ et je pose $x=1-\frac tn$ dans $\int_0^1 \frac {1-x^n}{1-x} dx$  ( Je te laisse trouver pour ne pas gâcher ton plaisir )
    Le 😄 Farceur


  • @Gebrane: Parce que tu crois que je ne commence pas de la même façon? >:)

    \begin{align}J&=\int_0^1 \left(\frac{1}{\ln x}+\frac{1}{1-x}\right)dx\\ &=\int_0^1 \left(\frac{1}{\ln x}+\frac{1-x^n}{1-x}+\frac{x^n}{1-x}+\frac{n}{1+nx}-\frac{n}{1+nx}\right)dx\\ &=\underbrace{\int_0^1 \left(\frac{1}{\ln x}+\frac{x^n}{1-x}+\frac{n}{1+nx}\right)dx}_{K_n}+\underbrace{\int_0^1 \frac{1-x^n}{1-x}dx}_{=H_n}-\underbrace{\int_0^1 \frac{n}{1+nx}dx}_{=\ln(1+n)}\\ K_n&=\underbrace{\int_0^{\frac{1}{n}}\left(\frac{1}{\ln x}+\frac{x^n}{1-x}+\frac{n}{1+nx}\right)dx}_{=K_{n,1}}+\underbrace{\int_{\frac{1}{n}}^1\left(\frac{1}{\ln x}+\frac{x^n}{1-x}+\frac{n}{1+nx}\right)dx}_{=K_{n,2}}\\ K_{n,1}&=\underbrace{\int_0^{\frac{1}{n}}\frac{1}{\ln x}dx}_{\rightarrow 0}+\underbrace{\int_0^{\frac{1}{n}}\frac{x^n}{1-x}dx}_{\rightarrow 0}+\underbrace{\int_0^{\frac{1}{n}}\frac{n}{1+nx}dx}_{=\ln 2}\\ \end{align}
  • Encore plus simple. (il manque les détails pour l'utilisation du théorème de convergence dominée)
  • Le deuxième lien est plus simple, je n' y avais pas pensé. .Pour le premier lien il faut un travail de plus pour éliminer les expo 
    Le 😄 Farceur


  • Fin de partie
    Modifié (March 2023)
    \begin{align*}
        a>0,H(a)&=\int_{\frac{1}{a}}^{a}\frac{1}{1+x^4}dx\\
        &\overset{u=\frac{1}{x}}=\int_{\frac{1}{a}}^{a}\frac{u^2}{1+u^4}du\\
        &=\frac{1}{2}\int_{\frac{1}{a}}^{a}\frac{1+u^2}{1+u^4}du\\
        &=\frac{1}{2}\int_{\frac{1}{a}}^{a}\frac{1+\frac{1}{u^2}}{2+\left(u-\frac{1}{u}\right)^2}du\\
        &\overset{w=u-\frac{1}{u}}=\frac{1}{2}\int_{\frac{1}{a}-a}^{a-\frac{1}{a}}\frac{1}{2+w^2}dw\\
        &=\frac{1}{\sqrt{2}}\arctan\left(\frac{a-\frac{1}{a}}{\sqrt{2}}\right)\\
        \lim_{a\rightarrow \infty}H(a)&=\int_0^\infty \frac{1}{1+x^4}dx=\lim_{a\rightarrow
            \infty}\frac{1}{\sqrt{2}}\arctan\left(\frac{a-\frac{1}{a}}{\sqrt{2}}\right)=\frac{\pi}{2\sqrt{2}} \end{align*}
    \begin{align*}J&=\int_{-1}^1 \frac{\arccos x}{1+x+x^2}dx\\
        &=\int_{0}^1 \frac{\frac{\pi}{2}+\arcsin x}{1-x+x^2}dx+\int_0^1 \frac{\arccos x}{1+x+x^2}dx\\
        &=\frac{\pi^2}{3\sqrt{3}}+\int_0^1 \frac{\arcsin x}{1-x+x^2}dx+\int_0^1 \frac{\arccos x}{1+x+x^2}dx\\
        &\overset{u=\frac{1-x}{1+x}}=\frac{\pi^2}{3\sqrt{3}}+\int_0^1 \frac{\pi-4\arctan(\sqrt{u})}{1+3u^2}du+\int_0^1 \frac{4\arctan(\sqrt{u})}{3+u^2}du\\
        &=\frac{2\pi^2}{3\sqrt{3}}-\int_0^1 \frac{4\arctan(\sqrt{u})}{1+3u^2}du+\int_0^1 \frac{4\arctan(\sqrt{u})}{3+u^2}du\\
        &=\frac{2\pi^2}{3\sqrt{3}}-4\int_0^1 \frac{\arctan(\sqrt{u})}{1+3u^2}du+4\left(\int_0^\infty \frac{\arctan(\sqrt{u})}{3+u^2}du-\underbrace{\int_1^\infty \frac{\arctan(\sqrt{u})}{3+u^2}du}_{z=\frac{1}{u}}\right)\\
        &=\frac{2\pi^2}{3\sqrt{3}}-4\left(\int_0^1 \frac{\arctan(\sqrt{u})}{1+3u^2}du+\int_0^1 \frac{\arctan\left(\frac{1}{\sqrt{z}}\right)}{1+3z^2}dz\right)+4\int_0^\infty \frac{\arctan(\sqrt{u})}{3+u^2}du\\
        &=4\int_0^\infty \frac{\arctan(\sqrt{u})}{3+u^2}du\\
    \end{align*}
    Pour $0\leq a\leq 1$, on définit $\displaystyle F(a)=\int_0^\infty \frac{\arctan(a\sqrt{u})}{3+u^2}du$
    Observer que, $F(0)=0,F(1)=\dfrac{J}{4}$, en sorte que,
    \begin{align*} \frac{J}{4}&=F(1)-F(0)=\int_0^1 F^\prime(a)da\\
    &=\int_0^1 \left(\int_0^\infty \frac{\sqrt{u}}{(1+a^2u)(3+u^2)}du\right)da\\
    &\overset{z=\sqrt{u}}=\int_0^1 \left(\int_0^\infty \frac{2z^2}{(1+a^2z^2)(3+z^4)}dz\right)da\\
    &\overset{\text{Décomp. élém. simples}}=\left(\underbrace{\int_0^\infty \frac{2z^2}{3+z^4}dz}_{x=\frac{3^{\frac{1}{4}}}{z}}\right)\left(\int_0^1 \frac{1}{1+3a^4}da\right)+\left(\underbrace{\int_0^\infty \frac{1}{3+z^4}dz}_{x=3^{-\frac{1}{4}}z}\right)\left(\int_0^1 \frac{6a^2}{1+3a^4}da\right)-\\&2\int_0^1 \frac{a}{1+3a^4}\Big[\arctan(az)\Big]_0^\infty da\\
    &=2\times 3^{-\frac{1}{4}}\left(\int_0^\infty \frac{1}{1+x^4}dx\right)\left(\underbrace{\int_0^1 \frac{1}{1+3a^4}da}_{w=3^{\frac{1}{4}}a}\right)+2\times 3^{\frac{1}{4}}\left(\int_0^\infty \frac{1}{1+x^4}dx\right)\left(\underbrace{\int_0^1 \frac{a^2}{1+3a^4}da}_{w=\frac{3^{-\frac{1}{4}}}{a}}\right)-\\&\pi\underbrace{\int_0^1 \frac{a}{1+3a^4}da}_{=\frac{\pi}{6\sqrt{3}}}\\
    &=\frac{\pi}{\sqrt{6}}\left(\int_0^{3^{\frac{1}{4}}}\frac{1}{1+w^4}dw+\int_{3^{-\frac{1}{4}}}^{\infty}\frac{1}{1+w^4}dw\right)-\frac{\pi^2}{6\sqrt{3}}\\
    &=\frac{\pi}{\sqrt{6}}\left(\int_{3^{-\frac{1}{4}}}^{3^{\frac{1}{4}}}\frac{1}{1+w^4}dw+\int_0^{\infty}\frac{1}{1+w^4}dw\right)-\frac{\pi^2}{6\sqrt{3}}\\
    &=\frac{\pi}{2\sqrt{3}}\arctan\left(\frac{3^{\frac{1}{4}}-3^{\frac{1}{4}}}{\sqrt{2}}\right)+\frac{\pi^2}{12\sqrt{3}}
    \end{align*}
    Ainsi,
    \begin{align*}
    \boxed{J=\frac{2\pi}{\sqrt{3}}\arctan\left(\frac{3^{\frac{1}{4}}-3^{-\frac{1}{4}}}{\sqrt{2}}\right)+\frac{\pi^2}{3\sqrt{3}}}
    \end{align*}
    NB: J'admets que,
    \begin{align*}
    &x\in [0,1],   
    \arcsin\left(\frac{1-x}{1+x}\right)=\frac{\pi}{2}-2\arctan\left(\sqrt{x}\right)\\
    &x\in [0,1],   
    \arccos\left(\frac{1-x}{1+x}\right)=2\arctan\left(\sqrt{x}\right)\\
    \end{align*}
  • gebrane
    Modifié (March 2023)
    Exploit 1
    Je n'ai jamais vu dans ma vie un calcul de $\displaystyle \int_0^{+\infty} \frac{1}{1+x^4} dx$ comme limite de $\displaystyle \int_{{1}/{a}}^{a} \frac{1}{1+x^4} dx$. Normalement, on procède par décomposition en éléments simples ou par résidus.
    Exploit 2
    Je n'aurais jamais pensé qu'on puisse transformer l'intégrale en $\arccos$ : $\displaystyle \int_{-1}^1 \frac{\arccos x}{1+x+x^2}dx$, en une intégrale avec $\arctan$ : $\displaystyle \int_0^\infty \frac{\arctan(\sqrt{u})}{3+u^2}du$."
    Défi
    Qui d'autre peut calculer $$\int_{-1}^1 \frac{\arccos x}{1+x+x^2}dx$$
    Je vais faire un essai ! 
    Le 😄 Farceur


  • Fin de partie
    Modifié (March 2023)
    @Gebrane: Le truc utilisé est souvent vu pour calculer $\displaystyle \int_0^\infty \frac{1}{1+x^4}dx$ mais en réalité il se généralise comme montré ci-dessus. Ce truc est quasi-magique quand on l'a vu une fois, on ne l'oublie pas.
    Je ne connaissais pas ou ne me souvenais pas de :
    \begin{align*} &x\in [0,1], \quad \arcsin\left(\frac{1-x}{1+x}\right)=\frac{\pi}{2}-2\arctan\left(\sqrt{x}\right)\\ &x\in [0,1], \quad \arccos\left(\frac{1-x}{1+x}\right)=2\arctan\left(\sqrt{x}\right)\\ \end{align*} Pour détecter ce type de formules, tu dérives puis tu vois la tête de la dérivée et tu intègres à nouveau si cela donne quelque chose d'intéressant tu cherches la constante manquante, ce qui est aisé dans les cas d'espèces.
    PS.
    Wolfy est capable de donner des primitives exploitables pour toutes les fractions rationnelles qui interviennent dans mon calcul, mais c'est moins joli.
  • Fin de partie est bien le champion du changement de variable dans les intégrales.
    Mais il s'est compliqué la vie pour passer de la première forme de $J$ à la seconde forme.
    Je reprends ses calculs en simplifiant un peu.

    1) Pour $x>0$ :  $\displaystyle\int_0^x\frac{1+t^2}{1+t^4}dt=\int_0^x\frac{d(t-1/t)}{(t-1/t)^2+2}=\frac1{\sqrt2}\left(\arctan\left(\frac{x^2-1}{x\sqrt2}\right)+\frac{\pi}2\right)$.
    On en déduit à l'aide de $t=\dfrac1u$ : $\quad I=\displaystyle\int_0^{+\infty}\frac{1}{1+t^4}dt=\int_0^{+\infty}\frac{t^2}{1+t^4}dt=\int_0^1\frac{1+t^2}{1+t^4}dt=\frac{\pi}{2\sqrt2}$.

    2) On effectue le changement de variable astucieux de Fin de partie directement dans l'intégrale $J$ : 
    $t=\dfrac{1-x}{1+x}\Leftrightarrow x=\dfrac{1-t}{1+t}$ donne $dx=\dfrac{-2dt}{(1+t)^2}$ et $x^2+x+1=\dfrac{3+t^2}{(1+t)^2}$
    d'où $J=\displaystyle\int_{-1}^1 \frac{\arccos x}{1+x+x^2}dx=2\int_0^{+\infty}\frac1{3+t^2}\arccos \frac{1-t}{1+t}dt=4\int_0^{+\infty}\frac{\arctan\sqrt t}{3+t^2}dt$
    en utilisant la formule connue : $\cos(2\theta)=\dfrac{1-\tan^2\theta}{1+\tan^2\theta}$.
    Il est pratique de poser $t=u^2\sqrt3$ pour obtenir $J=\dfrac8{\sqrt3}\displaystyle\int_0^{+\infty}\frac{u\arctan(3^{1/4}u)}{1+u^4}du$

    3) On introduit alors (pour $x>0$) $F(x)=\displaystyle\int_0^{+\infty}\frac{u\arctan(ux)}{1+u^4}du$  (on a aussi $F(0)=0$).
    $F'(x)=\displaystyle\int_0^{+\infty}\frac{u^2}{(1+u^4)(1+x^2u^2)}du=\dfrac1{1+x^4}\int_0^{+\infty}\left(\frac{u^2+x^2}{1+u^4}-\frac{x^2}{1+x^2u^2}\right)du$ (par décomposition en éléments simples).
    D'où $F'(x)=\dfrac{1+x^2}{1+x^4}I-\dfrac x{1+x^4}\dfrac{\pi}2=\dfrac{\pi}{2\sqrt2}\dfrac{1+x^2-x\sqrt2}{1+x^4}=\dfrac{\pi}{2\sqrt2}\dfrac1{1+x^2+x\sqrt2}$.

    Ensuite $F(x)=\dfrac{\pi}{\sqrt2}\displaystyle\int_0^x\frac1{1+(t\sqrt2)^2}dt=\dfrac{\pi}2(\arctan(x\sqrt2+1)-\pi/4)=\dfrac{\pi}2\arctan\dfrac x{x+\sqrt2}$.

    Pour $x=3^{1/4}$ cela donne $J=\dfrac{4\pi}{\sqrt3}\arctan\dfrac {3^{1/4}}{3^{1/4}+\sqrt2}$.
  • Fin de partie
    Modifié (March 2023)
    @Jandri: Merci, je me doutais que le début de mon calcul était trop compliqué.
    La solution de Tauraso qui n'est pas très différente de la mienne (il utilise la même identité pour arccos) mais fidèle à son habitude il utilise de la grosse artillerie pour faire certains calculs (Une version du théorème des résidus)
Connectez-vous ou Inscrivez-vous pour répondre.